Difference between revisions of "2018 AMC 12B Problems/Problem 15"
MRENTHUSIASM (talk | contribs) m |
MRENTHUSIASM (talk | contribs) m (Undo revision 163337 by MRENTHUSIASM (talk)) (Tag: Undo) |
||
Line 7: | Line 7: | ||
Let <math>\underline{ABC}</math> be one such odd positive <math>3</math>-digit integer with hundreds digit <math>A,</math> tens digit <math>B,</math> and ones digit <math>C.</math> Since <math>\underline{ABC}\equiv0\pmod3,</math> we need <math>A+B+C\equiv0\pmod3</math> by the divisibility rule for <math>3.</math> | Let <math>\underline{ABC}</math> be one such odd positive <math>3</math>-digit integer with hundreds digit <math>A,</math> tens digit <math>B,</math> and ones digit <math>C.</math> Since <math>\underline{ABC}\equiv0\pmod3,</math> we need <math>A+B+C\equiv0\pmod3</math> by the divisibility rule for <math>3.</math> | ||
− | As <math>A\in\{1,2,4,5,6,7,8,9\}</math> and <math>C\in\{1,5,7,9\},</math> there are <math>8</math> possibilities for <math>A</math> and <math>4</math> possibilities for <math>C.</math> Note that each ordered pair <math>(A,C)</math> determines the | + | As <math>A\in\{1,2,4,5,6,7,8,9\}</math> and <math>C\in\{1,5,7,9\},</math> there are <math>8</math> possibilities for <math>A</math> and <math>4</math> possibilities for <math>C.</math> Note that each ordered pair <math>(A,C)</math> determines the value of <math>B</math> modulo <math>3,</math> so <math>B</math> can be any element in one of the sets <math>\{0,6,9\},\{1,4,7\},</math> or <math>\{2,5,8\}.</math> We conclude that there are always <math>3</math> possibilities for <math>B.</math> |
By the Multiplication Principle, the answer is <math>8\cdot4\cdot3=\boxed{\textbf{(A) } 96}.</math> | By the Multiplication Principle, the answer is <math>8\cdot4\cdot3=\boxed{\textbf{(A) } 96}.</math> |
Revision as of 23:25, 8 October 2021
Contents
Problem
How many odd positive -digit integers are divisible by but do not contain the digit ?
Solution 1
Let be one such odd positive -digit integer with hundreds digit tens digit and ones digit Since we need by the divisibility rule for
As and there are possibilities for and possibilities for Note that each ordered pair determines the value of modulo so can be any element in one of the sets or We conclude that there are always possibilities for
By the Multiplication Principle, the answer is
~Plasma_Vortex ~MRENTHUSIASM
Solution 2
Let be one such odd positive -digit integer with hundreds digit tens digit and ones digit Since we need by the divisibility rule for
As and note that:
- There are possibilities for namely
There are possibilities for namely
There are possibilities for namely
- There are possibilities for namely
There are possibilities for namely
There are possibilities for namely
- There are possibility for namely
There are possibilities for namely
There are possibility for namely
We apply casework to Together, the answer is
~MRENTHUSIASM
Solution 3
Analyze that the three-digit integers divisible by start from In the 's, it starts from In the 's, it starts from We see that the units digits is and
Write out the - and -digit multiples of starting from and Count up the ones that meet the conditions. Then, add up and multiply by since there are three sets of three from to Then, subtract the amount that started from since the 's ll contain the digit
Together, the answer is
Solution 4
Consider the number of -digit numbers that do not contain the digit which is For any of these -digit numbers, we can append or to reach a desirable -digit number. However, we have and thus we need to count any -digit number twice. There are total such numbers that have remainder but of them contain so the number we want is Therefore, the final answer is
Solution 5
We need to take care of all restrictions. Ranging from to there are odd -digit numbers. Exactly of these numbers are divisible by which is Of these numbers, have in their ones (units) digit, have in their tens digit, and have in their hundreds digit. Thus, the total number of -digit integers is
~mathpro12345
Solution 6
We will start with the numbers that could work. This numbers include _ _ , _ _ , _ _ , _ _ . Let's work case by case.
Case : _ _ : The two blanks could be any number that is mod that does not include . We have cases for this case (we could count every case).
Case : _ _ : The blanks could be any number that is mod that does not include . But we could see that this case has exactly the same solutions to case because we have a correspondence. We can do the exact same for case .
Cases : _ _ : We need the blanks to be a multiple of , but does not contain 3. We have which also contains numbers. Therefore, we have which is equal to
~Arcticturn
Video Solution
https://youtu.be/mgEZOXgIZXs?t=448
~ pi_is_3.14
See Also
2018 AMC 12B (Problems • Answer Key • Resources) | |
Preceded by Problem 14 |
Followed by Problem 16 |
1 • 2 • 3 • 4 • 5 • 6 • 7 • 8 • 9 • 10 • 11 • 12 • 13 • 14 • 15 • 16 • 17 • 18 • 19 • 20 • 21 • 22 • 23 • 24 • 25 | |
All AMC 12 Problems and Solutions |
The problems on this page are copyrighted by the Mathematical Association of America's American Mathematics Competitions.